Logical expression using quantifiers

  • Thread starter Thread starter mutzy188
  • Start date Start date
  • Tags Tags
    Expression
Click For Summary
The discussion revolves around expressing the statement "There is no smallest positive real number" using logical quantifiers. One participant suggests the expression (∃y)((∀x)(y<x)) with the universe of discourse defined as positive real numbers. Another participant critiques this by stating it implies the existence of a smallest positive number, which contradicts the original statement. They propose a clearer expression: ∀x > 0, ∃y > 0: y < x, emphasizing the need to negate the existence of a smallest positive number. The conversation highlights the importance of clarity in logical expressions and the correct use of quantifiers.
mutzy188
Messages
37
Reaction score
0

Homework Statement



Express each statement as a logical expression using quantifiers. State the universe of discourse.:

There is no smallest positive real number

The Attempt at a Solution



(∃y)((∀x)(y<x) )x

universe of discourse: poaitive real numbers

Is this correct?

Thanks
 
Physics news on Phys.org
Why not just write: \forall x &gt; 0,\;\exists y &gt; 0:y &lt; x
 
Last edited:
Your statement says that there is a smallest positive number. You want to take the negation of that.
 
ZioX said:
Your statement says that there is a smallest positive number
No, my statement does not say that at all.

*Edit: If your post was directed at the OP, please state so in your post; else, your assertion is clearly false (your post comes directly after mine, so I assume you are referring to my post)
 
Last edited:
It is quite possible that ZioX wrote his reply before yours was posted- he thought his would appear immediately after mutzy188's post. Of course, you are right. It would have been clearer if he (and you- your reply might well have wound up after his) had copied the original post into the response.
 
Question: A clock's minute hand has length 4 and its hour hand has length 3. What is the distance between the tips at the moment when it is increasing most rapidly?(Putnam Exam Question) Answer: Making assumption that both the hands moves at constant angular velocities, the answer is ## \sqrt{7} .## But don't you think this assumption is somewhat doubtful and wrong?

Similar threads

  • · Replies 2 ·
Replies
2
Views
2K
  • · Replies 3 ·
Replies
3
Views
1K
  • · Replies 7 ·
Replies
7
Views
2K
  • · Replies 6 ·
Replies
6
Views
3K
  • · Replies 4 ·
Replies
4
Views
2K
  • · Replies 3 ·
Replies
3
Views
2K
  • · Replies 2 ·
Replies
2
Views
2K
  • · Replies 1 ·
Replies
1
Views
2K
  • · Replies 9 ·
Replies
9
Views
3K
  • · Replies 2 ·
Replies
2
Views
2K